What is Electromagetism: Definition and 282 Discussions

No Wikipedia entry exists for this tag
  1. C

    How to Optimize a simple Electromagnet

    My goal is to make an electromagnet that delivers 100mT magnetic field on one end of the iron. The iron core has a diameter 2.0mm and length 0.1m. I am using a copper wire that can carry 2.3A of current, and I am using a DC power supply. I have been able to create a 100mT magnetic field on the...
  2. S

    E-field of a moving charge at constant velocity

    I came upon this page that provides the answer: http://farside.ph.utexas.edu/teaching/em/lectures/node125.html But I can't figure out what gamma(u) is. Could anyone help?
  3. C

    Total magnetic moment of simple system.

    Homework Statement If there's a current I flowing through the rectangular conductor and it's located in the magnetic field in such way that normal vector of the surface that this rectangle forms closes the angle of 60 degrees with magnetic induction vector find total magnetic moment of this...
  4. Hanarchist

    Creating a Maglev Track using Electromagnets?

    Hi All, I was wondering how I would go about creating a circular maglev track using electromagnets? The idea is to have more control over it and to eventually write a piece of code that would allow changes in the strength of the magnetic field potentially? I was just wondering how it would work...
  5. C

    Two coils of wire with the same size and shape

    Homework Statement Two coils of wire that have same shape and dimensions are thickly rolled up so that the coincide, they only differ in number of coils (windings) N1 for the firs one and N2 for the second one (N1<N2). When there's constant current in the first one and there's no current in the...
  6. W

    Help using electromagnetic induction

    Homework Statement I am attempting to create a generator using electromagnetic induction. I am using a push pull system to move a bar magnet through a coil to create current. But I am unsure of the dimensions to build it with and when I attempt to use the equation I get stuck. I am looking for...
  7. M

    I Movement of a iron ring inside a toroidal solenoid

    What happens when you apply power to a toroidal solenoid with a iron ring inside? Does the ring move? Does the speed of movement depend on the amount of power? Sorry if this is too easy, I have no education in physic.
  8. G

    Divergence and transversal extension integral definitions

    Hi. I am reading a paper about gaussian beams and the author says that gaussian beams have simultaneously minimal divergence and minimal transversal extension. In order to prove it, the author states that \mathrm{divergenece} \propto \int_{-\infty}^{+\infty} \frac{d\,k_{x}}{2\pi}...
  9. V

    Having trouble conceptualizing capacitors

    So I basically understand the C = ε0(A/d) formula. Larger area means there is room for more electrons, smaller d means larger E field, which pulls electrons onto the opposite plate more strongly. What's tripping me up is when dielectrics come into the picture. When a dielectric is inserted, the...
  10. It's me

    How does ∇ × J = 0 relate to B = 0 in Maxwell's equations?

    Homework Statement Prove that a current density J(r, t) such that ∇ × J = 0 implies the magnetic field B = 0.Homework Equations Maxwell's equations, vector calculus The Attempt at a Solution I've played around with Maxwell's equations and with the properties of vector calculus but I can't...
  11. Syed Taha ahmed

    I Why does back EMF in DC machines deviate from a pure sine wave?

    Why voltages out of the rotating loop is constant positive and negative **Why nOT varying like sinusoidal **. Like it has a wave form of O to 2*vBL and then it maintain its value there for some time before failing to zero and changing direction. I have attach the image file from my book...
  12. P

    Confusion with electromagnetism and Faraday's law

    Hi- Sorry if this is a silly question, but by definition the magnetic flux is given by integral B dot dA. But From Gauss' law for magnetostatics is this not zero around a closed loop? So would that not then imply that the EMF around any closed loop is zero? Obviously I'm missing something, so I...
  13. C

    Force acting on conducting contour

    Homework Statement We have contours C1 and C2 located in vacuum and there are constant currents in them, I1 and I2 respectively. Find the expression for magnetic force that acts on one very small element (dl) of the circle C2 and it's coming from one very small element of the contour C1...
  14. S

    What governs how hard it is to turn a dynamo?

    Say I wanted to attach a dynamo up to some pedals or something - what factors would affect how hard the shaft would be to turn? Is it governed by the physical configuration of the dynamo (windings, etc)? Or is it the (electrical) load attached that matters - and how does that work? If I wanted...
  15. J

    I What is the resolution to the problem with E cross B field and E>B?

    I have been unable to find a satisfactory explanation of this problem, elsewhere. Consider an uniform electric field, E, along the y axis. Consider also a uniform magnetic field, B, along the z axis. If we release a particle (charge=q, mass=m) at rest on the origin at time t=0, what will be...
  16. M

    Second derivatives of magnetic potential

    Hi there! It looks like you are trying to prove that the second derivatives of the magnetic potential function ##\mathbf{A}## belong to the class ##C(\mathbb{R}^3)##. This is a great question and involves some advanced mathematical techniques. One approach you can take is to use the dominated...
  17. It's me

    Velocity of propagation of an EM field in vacuum

    Homework Statement In a region of empty space, the magnetic field is described by ##\vec{B} = B_0e^{ax}\sin{(ky-\omega t)} \hat{z}##. Find the speed of propagation ##\vec{v}## of this field. Homework Equations ##\Delta \vec{B} = \frac{1}{v^2}\frac{d^2\vec{B}}{dt^2}## , ##k=\frac{\omega }{...
  18. DavideGenoa

    Commutations and delta in deriving Ampère's law

    Hi, friends! I have been able to understand, thanks to Hawkeye18, whom I thank again, that, if ##\mathbf{J}## is measurable according to the usual ##\mathbb{R}^3## Lebesgue measure ##\mu_{\mathbf{l}}## and bounded, a reasonable hypothesis if we consider it the density of current, if...
  19. A

    I need to create a .7+ Tesla electromagnetic solenoid

    I am trying to create a solenoid with a high flux density, but I need the specifications (wire gauge, current applied, etc). The diameter of the core would be preferable around 15mm. The length can be pretty much anything reasonable. I need this for a project that requires a strong repulsive...
  20. C

    Superconductive Contour with Inductance and Magnetic Field: Current Calculation

    1. The problem statement, all variables and given/known data We have a superconductive contour in the shape of circle with radius ##a##. Inductance of contour is ##L##, when the contour is out of magnetic field, there's no current in it. What's the current in the contour when constant magnetic...
  21. xareu

    Work done by permanent magnets

    I recall that the energy stored in a magnetic field is half the volume integral of the dot product of magnetic field and flux density vectors (H and B). When you place a piece of iron near a magnet, it does a work by attracting it. If H is 0 (I think there is only the magnetic moment vector M...
  22. S

    Electromotive force in a moving bar

    Hi everyone, I've seen in some lecture notes that a moving conducting bar (length of L) in a uniform and constant magnetic field develops an electromotive force (emf) between the 2 edges of the bar. Lets assume that the bar moves to the right (with velocity v) and the magnetic field (B) points...
  23. T

    Force on magnet in magnetic field

    Hello, i've recently found the post "How to calculate the force acting on a magnet placed in a magnetic field?" at http://physics.stackexchange.com/questions/20258/how-to-calculate-the-force-acting-on-a-magnet-placed-in-a-magnetic-field in another Forum and the answer said smth about an...
  24. DavideGenoa

    Biot-Savart law from Ampère's (with multivariate calculus)

    Let us assume the validity of Ampère's circuital law\oint_{\gamma}\mathbf{B}\cdot d\mathbf{x}=\mu_0 I_{\text{linked}}where ##\mathbf{B}## is the magnetic field, ##\gamma## a closed path linking the current of intensity ##I_{\text{linked}}##. All the derivations of the Biot-Savart law for a...
  25. S

    What is q and what is the charge (negative or positive)?

    Three identical point charges, each of mass m 0.10kg and charge q, hang from three strings, as pictured below. The lengths of the left and right strings are L 30.0cm and the angle is 45.0o. Just on looking at the picture I think the charge is positive since there is tension separation the...
  26. V

    Has the Charge Quantization Problem Been Solved Without Magnetic Monopoles?

    I am no mathematician, not even an expert in Gauge Theories, but I came across this article http://arxiv.org/abs/1409.6716 (published here http://www.sciencedirect.com/science/article/pii/S0393044015002284) when I was looking for some "condensed-matter-type-monopoles", and the author claims...
  27. J

    Increase Phase Velocity by Losing Power ?

    I just read some words from the book Optical Waveguide Theory by Snyder, Allan W., p226. It says that "However, although it is possible for the phase velocity of a mode to exceed the maximum speed of light in the cladding (of waveguide), c/n_cl, this cannot occur without losing power to...
  28. A

    Electromagnetic induction by a rotating sphere

    Homework Statement http://physics.columbia.edu/files/physics/content/Quals2010Sec2.pdf Problem 1 Consider a rigid, ideally conducting sphere of radius R, with total charge zero. The sphere rotates with angular velocity Ω, ΩR<<c. Suppose a dipole magnetic field threads the sphere. the dipole is...
  29. Shailesh Pincha

    Electric & Magnetic Fields Out of Phase: Why?

    Why can't we have an Electromagnetic wave in which the Electric Field and Magentic Field aren't in phase?
  30. tomdodd4598

    Electromagnetic Tensor in (-+++) Convention

    Hi there, Over the last couple of weeks, I have been learning about the relativistic description of electromagnetism through Leonard Susskind's Theoretical Minimum lectures, and although I have managed to follow it, there are some parts which I am becoming increasingly confused by, not helped...
  31. Ryan Reed

    Do Electrons need Energy to Excite the EM field?

    Electrons have both mass and electromagnetic charge, so why is it that an electron's rest energy is equal to its mass energy with E=mc^2? Shouldn't it have some energy left over to excite the electromagnetic field? The mass energy excites the Higgs field, so why is there no energy for the EM field?
  32. LarryS

    Classical vs Quantum Electromagetism

    I'm having trouble relating the classical (Maxwell) model for electromagnetic radiation to the quantum mechanical (Q.E.D.) model. How do the magnetic field wave and electic field wave components of the classical electromagnetic wave relate to the quantum mechanical wave function for a photon?
Back
Top